1answer.
Ask question
Login Signup
Ask question
All categories
  • English
  • Mathematics
  • Social Studies
  • Business
  • History
  • Health
  • Geography
  • Biology
  • Physics
  • Chemistry
  • Computers and Technology
  • Arts
  • World Languages
  • Spanish
  • French
  • German
  • Advanced Placement (AP)
  • SAT
  • Medicine
  • Law
  • Engineering
pychu [463]
3 years ago
12

Consider the hypotheses shown below. Given that x=117

Mathematics
1 answer:
motikmotik3 years ago
3 0

Solution: We are given:

\bar{x}=117, \sigma=25, n=48, \alpha=0.01

The hypotheses under consideration is:

H_{0}:\mu=123

H_{A}:\mu\neq123

First let's find the critical value at \alpha=0.01.

Since the alternative hypothesis is two tailed, therefore we will have two critical values.

Also, since the population standard deviation is given, therefore we will use standard normal distribution to find the critical value.

Using the standard normal table, the critical values are:

z_{critical}=-2.58,2.58

Therefore, decision rule in terms of the critical values of the test statistic is:

Reject the null hypothesis if the calculated value of the test statistic, z is not contained within the critical values, -2.58,2.58. Otherwise do not reject the null hypothesis.

You might be interested in
2x-32=12<br> means they spent how much
Hoochie [10]

Answer:

22

Step-by-step explanation:

6 0
2 years ago
What is the perimeter of 6*4
Diano4ka-milaya [45]
24 would be the answer if its wrong i am so sorry but it has to be right

6 0
3 years ago
What type of polynomial is: 3x+x^2+4 <br>A.quadratic<br> B. quartic<br> C. linear <br>D. cubic
Svetradugi [14.3K]

Answer:

A.quadratic

Step-by-step explanation:

3x+x^2+4

Put in order from largest to smallest power of x

x^2 +3x+4

The highest power is 2

A.quadratic   highest power 2

B. quartic   highest power 4

C. linear  highest power 1

D. cubic highest power 3

3 0
2 years ago
Write an equation in standard form for each line.
LenaWriter [7]

The standard form: Ax + By = C.

The point-slope form:

y-y_1=m(x-x_1)\\\\m=\dfrac{y_2-y_1}{x_2-x_1}


7. We have the points (4, -7) and (2, -3). Substitute:

m=\dfrac{-3-(-7)}{2-4}=\dfrac{4}{-2}=-2\\\\y-(-7)=-2(x-4)\qquad|\text{use distributive property}\\\\y+7=(-2)(x)+(-2)(-4)\\\\y+7=-2x+8\qquad|-7\\\\y=-2x+1\qquad|+2x\\\\\boxed{2x+y=1}

8. We have the points (1, 5) and (-10, -6). Substitute:

m=\dfrac{-6-5}{-10-1}=\dfrac{-11}{-11}=1\\\\y-5=1(x-1)\\\\y-5=x-1\qquad|+5\\\\y=x+4\qquad|-x\\\\-x+y=4\qquad|\text{change the signs}\\\\\boxed{x-y=-4}


7 0
3 years ago
Help,please<br> It’s calculus
NikAS [45]

The function is a continuous function at 2 if the LHL and RHL will be the same as the limit of the function.

<h3>What is continuity of a function?</h3>

It is defined as the property of a function in which the function varies continuous, and we plot the graph of a function it doesn't break.

We have a function:

\rm f(x) = \left \{ {{x^2-x}, x\leq 2\atop {2x - 2 \ ,x > 2 \  } \right.

If f(x) is a continuous, then it will follow:

f(2) = 2² - 2 = 4 - 2 = 2

Left-hand limit at 2 = right-hand limit at 2 = 2

Limit at 2 of a function = 2

Using limit, we can check the whether the function is differentiable or not.

Thus, the function is a continuous function at 2 if the LHL and RHL will be the same as the limit of the function.

Learn more about the continuous function here:

brainly.com/question/21447009

#SPJ1

4 0
2 years ago
Other questions:
  • (47) Help check my geo ans pls
    13·1 answer
  • Eight subtracted from a number is twelve
    6·2 answers
  • Need help! Nd please write down steps. Thank u
    12·2 answers
  • Ashley is preparing for a horse riding competition. She has trained her horse for 5 hours and has completed 165 rounds. At what
    10·1 answer
  • What is the acronym used to describe the order of operations?
    10·1 answer
  • How many 1/8s are in 3/4s
    8·2 answers
  • Hey! Can someone please help me with this question? Really appreciate it
    13·1 answer
  • Which point in the coordinate plane represents the number 7+5i in the complex plane?
    14·1 answer
  • 5. Which sequence of transformations maps quadrilateral ABCD onto quadrilateral EFGH?​
    15·2 answers
  • In ΔRST, = 93 inches, ∠S=123° and ∠T=28°. Find the length of r, to the nearest 10th of an inch
    8·1 answer
Add answer
Login
Not registered? Fast signup
Signup
Login Signup
Ask question!